Geomania.Org Forumları

Fantezi Cebir => Fantezi Cebir => Konuyu başlatan: ERhan ERdoğan - Mayıs 04, 2015, 01:21:18 öö

Başlık: 20. Ulusal Antalya Matematik Olimpiyatı Soruları
Gönderen: ERhan ERdoğan - Mayıs 04, 2015, 01:21:18 öö
...
Başlık: Ynt: 20. Ulusal Antalya Matematik Olimpiyatı Soruları
Gönderen: ERhan ERdoğan - Mayıs 04, 2015, 05:47:25 ös
6.

$T = a^3+b^3+c^3+d^3 = a^3+\dfrac{b^3}{2}+\dfrac{b^3}{2}+\dfrac{c^3}{2}+\dfrac{c^3}{2}+d^3$

$T_{1} =  a^3+\dfrac{b^3}{2}+\dfrac{c^3}{2}$  ve  $T_{2} =\dfrac{b^3}{2}+\dfrac{c^3}{2}+d^3$ diyelim.

AGO eşitsizliğinden,

$T_{1} \geq \dfrac{3abc}{\sqrt[3]{4}}$  ve  $T_{2} \geq \dfrac{3bcd}{\sqrt[3]{4}}$ dir.

$T = T_{1}+T_{2} \geq \dfrac{3}{\sqrt[3]{4}}(abc+bcd)$

$S = \dfrac{abc+bcd}{a^3+b^3+c^3+d^3} \leq \dfrac{\sqrt[3]{4}}{3}$  bulunur.
Başlık: Ynt: 20. Ulusal Antalya Matematik Olimpiyatı Soruları
Gönderen: Eray - Mayıs 05, 2015, 02:05:55 öö
5.

Sayılar $\sqrt{\dfrac{x}{12}}+\sqrt{\dfrac{108}{x}}$ formundadır.


A.G.O eşitsizliğinden,

$\sqrt{\dfrac{x}{12}}+\sqrt{\dfrac{108}{x}} \ge 2\sqrt{\sqrt{\dfrac{108}{12}}}=2\sqrt{3}$

Yani sayılardan en küçüğü $2\sqrt3$ tür.

Eşitlik durumu $\Longrightarrow \dfrac{x}{12}=\dfrac{108}{x} \Longrightarrow x=36$ için sağlanır, ve bu da sayılarımıza dahildir. Dolayısıyla cevap $2\sqrt3$ tür.
Başlık: Ynt: 20. Ulusal Antalya Matematik Olimpiyatı Soruları
Gönderen: onokumus - Mayıs 05, 2015, 02:26:09 öö
5.

Sayılar $\sqrt{\dfrac{x}{12}}+\sqrt{\dfrac{108}{x}}$ formundadır.


A.G.O eşitsizliğinden,

$\sqrt{\dfrac{x}{12}}+\sqrt{\dfrac{108}{x}} \ge 2\sqrt{\sqrt{\dfrac{108}{12}}}=2\sqrt{6}$

Yani sayılardan en küçüğü $2\sqrt6$ dır.

Eşitlik durumu $\Longrightarrow \dfrac{x}{12}=\dfrac{108}{x} \Longrightarrow x=36$ için sağlanır, ve bu da sayılarımıza dahildir. Dolayısıyla cevap $2\sqrt6$ dır.
Sanırım işlem hatası yapmışsınız. Cevap: $2\sqrt3$ olacak.
Başlık: Ynt: 20. Ulusal Antalya Matematik Olimpiyatı Soruları
Gönderen: mehmetutku - Mayıs 05, 2015, 12:10:52 ös
17.

Açıortay teoreminden $|AP|=3x$  ise  $|AB|=2x+2$  olur.   $AO$  nun çemberi kestiği nokta $K$  olsun.  $\angle PAO=\alpha$ dersek kirişler dörtgeninden $\angle CBK=\alpha$ olur. Çapı gördüğü için $\angle ABK=90^\circ$  dir.  Dolayısıyla $\angle ABD=\angle APO$  olur.  Bu da $\triangle ABD \cong \triangle APD$   demektir.   O zaman $|AB|=|AP|$  olur.  $2x+2=3x$  ve  $x=2$  den  $|AP|=6$   bulunur.
Başlık: Ynt: 20. Ulusal Antalya Matematik Olimpiyatı Soruları
Gönderen: mehmetutku - Mayıs 05, 2015, 12:16:14 ös
21.

$\angle BAH=15^\circ$  ve  $\angle HAE=35^\circ$  dir.  $ABDC$  kirişler dörtgeni olduğu için  $\angle ABC=\angle ADC=75^\circ$  olur.  $HDCE$  kirişler dörtgeni olduğundan $\angle HDC=\angle HEA=75^\circ$  olur. $\triangle HEA$  da $\angle AHE=180-35-75=70^\circ$  bulunur.
Başlık: Ynt: 20. Ulusal Antalya Matematik Olimpiyatı Soruları
Gönderen: mehmetutku - Mayıs 05, 2015, 12:26:38 ös
1.

Sorudaki ilk denklemi düzenlersek $x^{x+y}=y^{x-y}$  bulunur.  İkinci eşitlik yerine yazılırsa  $x^{x+\tfrac{1}{x^2}}=(\dfrac{1}{x^2})^{x-\tfrac{1}{x^2}}$   bulunur.  Düzenlersek  $x^{3x-\tfrac{1}{x^2}}=1$   olur. Burada iki durum vardır. $x=1$ için $y=1$  çözümü gelir. $3x-\tfrac{1}{x^2}=0$  için $x=\sqrt[3]{\dfrac{1}{3}}$    ve  $y=\sqrt[3]{9}$  çözümü  gelir.  Yani toplam $2$  çözüm vardır.
Başlık: Ynt: 20. Ulusal Antalya Matematik Olimpiyatı Soruları
Gönderen: mehmetutku - Mayıs 05, 2015, 12:53:27 ös
7.

$n=1$  için $a_1=2$

$n=2$  için $a_2=\dfrac{6}{5}$

İddaamız $a_n=1+\dfrac{1}{5^{n-1}}$   olduğudur..  Bunu tümevarım ile gösterelim. $n=1$  için doğrudur. $n$  için doğru olsun.
O zaman $5a_{n+1}=1+\dfrac{1}{5^{n-1}}+4$    ve  $a_{n+1}=1+\dfrac{1}{5^n}$  dir.  Yani  iddaamız doğruymuş.

Soruda tanımladığı  $A_n=a_1+a_2+ \cdots +a_n= 1+\dfrac{1}{5^0}+1+\dfrac{1}{5^1}+\cdots+1+\dfrac{1}{5^{n-1}}=n+1+\dfrac{1-\dfrac{1}{5^n}}{1-\dfrac{1}{5}}=n+1+\dfrac{5^n-1}{4\cdot5^{n-1}}$   olur.

$A_n-n-\dfrac{5}{4}=n+1+\dfrac{5^n-1}{4\cdot5^{n-1}}-n-\dfrac{5}{4}=\dfrac{5^n-1}{4\cdot5^{n-1}}-\dfrac{1}{4}=-\dfrac{1}{4\cdot5^{n-1}}$    olur.

Mutlak değerden çıkarırsak $\dfrac{1}{4\cdot5^{n-1}} \lt \dfrac{1}{2500}$  eşitsizliğini sadeleştirirsek $5^{4} \lt 5^{n-1}$   ve  $5\lt n$  bulunur.  Dolayısıyla  $n$  nin alabileceği en küçük değer  $6$  dır.
Başlık: Ynt: 20. Ulusal Antalya Matematik Olimpiyatı Soruları
Gönderen: mehmetutku - Mayıs 05, 2015, 02:54:40 ös
2.

Terimler $\dbinom {1515}{a}\cdot3^{\tfrac{1515-a}{3}}\cdot5^{\tfrac{a}{5}}$   formundadır. Terimlerin rasyonel olması için  $3$  ve  $5$  in $a$  yı bölmesi gerekmektedir.  $0$  dan  $1515$  e kadar $15$  e bölünen  $102$  sayı vardır.
Başlık: Ynt: 20. Ulusal Antalya Matematik Olimpiyatı Soruları
Gönderen: mehmetutku - Mayıs 05, 2015, 03:13:02 ös
3.

$(a+b+c+d)^n$  deki terim sayısı dağılım formülünden  $\dbinom {n+4-1}{4-1}=\dbinom {n+3}{3}$  şeklindedir.  Ayrıca $\dbinom {n}{a}=\dbinom{n+1}{a+1}-\dbinom {n}{a+1}$ 

olduğunu biliyoruz.  O zaman  $f(2n)-f(2n-1)=\dbinom {2n+3}{3}-\dbinom {2n+2}{3}=\dbinom{2n+2}{2}$   dir. Bizden istenen ifade

$\dbinom {4}{2}\cdot \dbinom{6}{2}\cdots \dbinom{102}{2}=\dfrac{102\cdot101\cdots 4\cdot3}{2^{50}}=\dfrac{102\cdot101\cdots 4\cdot3\cdot2}{2^{51}}=\dfrac{102!}{2^{51}}$  dir.  $102!$  in içinde $98$  tane $2$  çarpanı vardır.  Dolayısıyla

cevap $98-51=47$   olarak bulunur.
Başlık: Ynt: 20. Ulusal Antalya Matematik Olimpiyatı Soruları
Gönderen: mehmetutku - Mayıs 05, 2015, 06:56:35 ös
19.

$a \triangle (b \triangle b)=a \triangle 0=a \triangle b +b$  tür.

$a \triangle (a \triangle a)= a \triangle 0 = a \triangle a + a=a$  dır.

İki denklemi birleştirirsek  $a \triangle b =a-b$  çıkar.  Cevap $(31 \triangle 13) \triangle 7=18 \triangle 7=11$  olarak bulunur.
Başlık: Ynt: 20. Ulusal Antalya Matematik Olimpiyatı Soruları
Gönderen: mehmetutku - Mayıs 05, 2015, 07:05:44 ös
8.

Dikdörtgeni $6\times 8$ lik dikdörtgene ve sağda bir sütun ve aşağıda bir satır olacak şekilde ayıralım. $6\times 8$  lik dikdörtgeni nasıl doldurursak dolduralım sağdaki sütun ve aşağıdaki satırın dizilimi tek şekilde olur. Mesela $6\times 8$  lik dikdörtgenin ilk satırı $7,9,7,9,9,9,9,7$  olsun. Sağda kalan sütunun o satırla kesiştiği karedeki sayı $7$ olmak zorundadır. Yani cevap  $2^{48}$  olur. Bu sayının da pozitif bölenlerinin sayısı $49$  dur.
Başlık: Ynt: 20. Ulusal Antalya Matematik Olimpiyatı Soruları
Gönderen: Eray - Mayıs 05, 2015, 07:24:20 ös
24.

Her $x$ için $x^2+ax+10b\ge0$ olması, denklemin diskriminantının $0$ dan küçük eşit olduğunu gösterir.
$\Longrightarrow 0\ge a^2-40b \Longrightarrow b\ge\dfrac{a^2}{40}$

$\Longrightarrow \dfrac{b+11}{a-1} \ge \dfrac{\dfrac{a^2}{40}+11}{a-1} = \dfrac{a^2+440}{40(a-1)} = \dfrac{1}{40}\cdot \dfrac{a^2+440}{a-1} = \dfrac{1}{40} \left(a-1 + \dfrac{441}{a-1}+2\right)$

Soruda $a>1$ verildiğinden A.G.O Eşitsizliği gereği, $a-1 + \dfrac{441}{a-1} \ge 42$ dir.

O halde $\dfrac{b+11}{a-1} \ge \dfrac{1}{40} \left(a-1 + \dfrac{441}{a-1}+2\right) \ge \dfrac{1}{40}(42+2) = \dfrac{11}{10}$ bulunur.

Eşitlik durumu $a-1 = \dfrac{441}{a-1} \Longrightarrow a=21, b=\dfrac{a^2}{40}=\dfrac{441}{40}$ iken sağlanır.
Başlık: Ynt: 20. Ulusal Antalya Matematik Olimpiyatı Soruları
Gönderen: mehmetutku - Mayıs 06, 2015, 10:28:16 öö
18.

$n$  metrelik kuyudan kaymadan çıkma sayısı $a_n$  olsun. Eğer $n-1$ metreye çıkarsa geri kalan metreyi tek şekilde çıkar. Yani $a_{n-1}$  durum.  Ve eğer $n-2$ metreye çıkarsa geri kalan iki metreyi de 2 zıplayarak tek şekilde çıkar. Çünkü 1 metre 1 metre zıplaması bir önceki durum oluyor. Yani $a_{n-2}$  durum.  Buradan $a_n=a_{n-1}+a_{n-2}$  elde edilir.

$a_1=1 \ , \ a_2=2 \ , \ a_3=3 \ , \ a_4=5 \ , \ a_5=8 \ , \ a_6=13 \ , \ a_7=21 \ , \ a_8=34 \ , \ a_9=55 \ , \ a_{10}=89$

Şimdi kurbağanın kayma durumlarını sayalım. Eğer birinci metreden sonra kayarsa $a_1 \cdot a_{10}$

İkinci metreden sonra kayarsa  $a_2 \cdot a_9$

Üçüncü metreden sonra kayarsa  $a_3 \cdot a_8$

Yani cevabımız $a_1 \cdot a_{10}+a_2 \cdot a_9+a_3 \cdot a_8+a_4 \cdot a_7+a_5 \cdot a_6+a_6 \cdot a_5+a_7 \cdot a_4+a_8 \cdot a_2+a_9 \cdot a_1+a_{10} \cdot a_1=1020$   bulunur.
Başlık: Ynt: 20. Ulusal Antalya Matematik Olimpiyatı Soruları
Gönderen: mehmetutku - Mayıs 06, 2015, 08:12:21 ös
20.

$f(1-k)$  yı düzenleyerek yazalım.

$f(1-k)=(1-k)^2\cdot (1-k)^4+(1-k^2)\cdot (1-k)^4+(2k+1)\cdot (1-k)^4+(3k+1)\cdot(1-k)^3+(4k+1)\cdot(1-k)^2+(5k+1)\cdot (1-k)+6k+14$

$f(1-k)=(1-2k+k^2+1-k^2+2k+1)\cdot (1-k)^4+(3k+1)\cdot(1-k)^3+(4k+1)\cdot(1-k)^2+(5k+1)\cdot (1-k)+6k+14$

$f(1-k)=3\cdot(1-k)^4+(3k+1)\cdot(1-k)^3+(4k+1)\cdot(1-k)^2+(5k+1)\cdot (1-k)+6k+14$

Yine düzenleyelim.

$f(1-k)=3\cdot(1-k)^2\cdot(1-k)^2+(3k+1)\cdot(1-k)\cdot(1-k)^2+(4k+1)\cdot(1-k)^2+(5k+1)\cdot (1-k)+6k+14$

$f(1-k)=(3-6k+3k^2+3k-3k^2+1-k+4k+1)\cdot(1-k)^2+(5k+1)\cdot (1-k)+6k+14$

$f(1-k)=5\cdot(1-k)^2+(5k+1)\cdot (1-k)+6k+14$

$f(1-k)=5-10k+5k^2+5k-k^2+1-k+6k+14$

$f(1-k)=20$

Yani $44-2k=20$  den  $k=2$  dir. O zaman

$f(x)=x^6+3x^5+5x^4+7x^3+9x^2+11x+26$

$f(1)=62$  bulunur.
Başlık: Ynt: 20. Ulusal Antalya Matematik Olimpiyatı Soruları
Gönderen: mehmetutku - Mayıs 06, 2015, 08:33:07 ös
25.

Cauchy Schwarz eşitsizliğinden

$(\dfrac{|BC|}{x}+\dfrac{|AC|}{y}+\dfrac{|AB|}{z})\cdot(|BC|\cdot x+|AC|\cdot y+|AB|\cdot z) \ge (|BC|+|AC|+|AB|)^2$     dir.

$|BC|\cdot x+|AC|\cdot y+|AB|\cdot z$   ifadesi üçgenin alanının iki katına eşittir. Yani sabittir. Ayrıca  $(|BC|+|AC|+|AB|)^2$   ifadesi de üçgenin çevresinin karesine eşittir. Yani o da sabittir. O zaman $\dfrac{|BC|}{x}+\dfrac{|AC|}{y}+\dfrac{|AB|}{z}$  ifadesinin minimum olması için eşitlik durumu olmalıdır.   Bu eşitsizliğin eşitlik durumu da   $\dfrac{\dfrac{|BC|}{x}}{|BC|\cdot x}=\dfrac{\dfrac{|AC|}{y}}{|AC|\cdot y}=\dfrac{\dfrac{|AB|}{z}}{|AB|\cdot z}$  olduğu zaman yani   $\dfrac{1}{x^2}=\dfrac{1}{y^2}=\dfrac{1}{z^2}$   olduğunda olur.  Bu da  $x=y=z$    demektir  yani  $P$  üçgenin iç teğet çemberinin merkezidir.
Başlık: Ynt: 20. Ulusal Antalya Matematik Olimpiyatı Soruları
Gönderen: mehmetutku - Mayıs 06, 2015, 08:42:29 ös
11.

A nın sayılarına göre durumlara ayıralım.

$0$ tane A  varsa $\Rightarrow \dbinom {9}{0}\cdot2^9=512$

$2$ tane A  varsa $\Rightarrow \dbinom {9}{2}\cdot2^7=4608$

$4$ tane A  varsa $\Rightarrow \dbinom {9}{4}\cdot2^5=4032$

$6$ tane A  varsa $\Rightarrow \dbinom {9}{6}\cdot2^3=672$

$8$ tane A  varsa $\Rightarrow \dbinom {9}{8}\cdot2^1=18$

Cevap : $512+4608+4032+672+18=9842$  bulunur.
Başlık: Ynt: 20. Ulusal Antalya Matematik Olimpiyatı Soruları
Gönderen: mehmetutku - Mayıs 06, 2015, 09:50:02 ös
4.

$B$  köşesine  $(0,0)$  noktası diyelim.  Ve üçgeni karmaşık sayılar düzlemine yerleştirelim.  $A$  köşesi $(a+15i)$   ve  $C$  köşesi de  $(b+3i)$  olsun.

Teorem: $\angle ABC=\alpha$  ve $A=(a+ci)$  ve  $C=(b+di)$  ise  $(a+ci)\cdot (\cos \alpha+i\cdot \sin \alpha)=(b+di)$   dir.

O zaman  $(a+15i)(\cos 60+i\cdot \sin 60)=(b+3i)$

$(a+15i)(\dfrac{1}{2}+i\cdot\dfrac{\sqrt{3}}{2})=(b+3i)$

$\dfrac{a-15\sqrt{3}}{2}+i\cdot \dfrac{15+a\sqrt{3}}{2}=b+3i$

$\dfrac{a-15\sqrt{3}}{2}=b$   ve   $ \dfrac{15+a\sqrt{3}}{2}=3$

$a=-3\sqrt{3}$   ve   $b=-9\sqrt{3}$   olur.   Bir karmaşık sayının ($a+bi$) orjine olan uzaklığı  $\sqrt{a^2+b^2}$  olduğu için  $A$  köşesinin  $B$  köşesine uzaklığı  $\sqrt{(-3\sqrt{3})^2+15^2}=\sqrt{252}$   bulunur.  Yani eşkenar üçgenin bir kenar uzunluğunu bulduk. Bu üçgenin alanı ise $\dfrac{252\sqrt{3}}{4}={63\sqrt{3}}$   bulunur.
Başlık: Ynt: 20. Ulusal Antalya Matematik Olimpiyatı Soruları
Gönderen: taftazani44 - Mayıs 07, 2015, 10:19:16 ös
4. Soru AC uzatılıp benzerlik, dik üçgen ve Cos teo.den de olur.
Başlık: Ynt: 20. Ulusal Antalya Matematik Olimpiyatı Soruları
Gönderen: cersoy - Mayıs 11, 2015, 12:41:04 ös
 
x in rasyonel olması durumunda hem c hemde  d irrasyonel olacaktır.oysa sadece bir sayı irrasyoneldir.
o halde x irrasyonel olmak zorundadır.buna göre;

a+b=4x/3-1/x+1/x-x/3=x

 Bundan dolayı,a ve b den en az biri irrasyoneldir.O halde,c ve d rasyoneldir.
c=x-√(3 )  ∈Q olur.x=c+√3   dir.d=(c+√3)^2-3√3∈Q

d=c^2+2c√3+3-3√3  ∈Q ise 2c=-3 olur.
d=(-3/2)^2+3=21/4
Başlık: Ynt: 20. Ulusal Antalya Matematik Olimpiyatı Soruları
Gönderen: Lokman Gökçe - Mayıs 13, 2015, 02:00:51 ös
10. Cevap $\boxed{B}$

$-1,1,2,3$ sayılarından sırasıyla $a,b,c,c$ tane bulunsun. $-a+b+5c=5$ ve $a+b+97c=995$ dir. $T=x_1^5 +x_2^5 + \dots +x_n^5$ olsun. $T=275c$ olduğundan $c$ nin değerini minimum seçtiğimizde $T$ ifadesi de minimum olur. Yani $c=1$ için $T_\min =275$ elde edilir. Bu halde $a=b$ ve $a+b=898$ dir. $n=a+b+c+c = 449 +449 + 1 + 1 = 900$ bulunur.
Başlık: Ynt: 20. Ulusal Antalya Matematik Olimpiyatı Soruları
Gönderen: Lokman Gökçe - Mayıs 13, 2015, 02:11:31 ös
13.

$\dfrac{1}{n+1}<(\sqrt5 - 2)^4 < \dfrac{1}{n}$ eşitsizliğini $n<(\sqrt5 + 2)^4 < n+1$ biçiminde ifade edelim. $a,b$ pozitif tamsayılar olmak üzere $(\sqrt5 + 2)^4 = a + b\sqrt5$ biçimindedir.

Dolayısıyla $ (\sqrt5 - 2)^4 = a - b\sqrt5$ olur. Üstelik $ 0< \sqrt5 - 2<1$ olduğundan $ 0< (\sqrt5 - 2)^4<1$ dir. Böylece

$ (\sqrt5 + 2)^4 + (\sqrt5 - 2)^4 = 2a $ ve $ 2a-1 < (\sqrt5 + 2)^4 < 2a $ olur.

$ (\sqrt5 + 2)^4 + (\sqrt5 - 2)^4 = 2\cdot (5^2 + 6\cdot 5\cdot 2^2 + 2^4) = 322$ elde edilir. $n=321$ bulunur.
Başlık: Ynt: 20. Ulusal Antalya Matematik Olimpiyatı Soruları
Gönderen: muuurat - Temmuz 07, 2015, 01:16:42 ös
15.
a$\leq$b$\leq$c olsun.a=$\frac{5-bc}{b+c-bc}$dir.a pozitif tamsayı olduğundan 5$\geq$b+c olmalıdır.a=1 için b+c=5.......(a,b,c)=(1,1,4)$\mapsto$3 tane ,      (a,b,c)=(1,2,3)$\mapsto$3!=6 tane. a,b ve c den ikisi tek sayı olmak zorundadır.a=2 olduğunda b ve c tek sayı olduğundan b+c toplamı 5 i geçeceğinden çözüm bitmiştir.Toplam 3+6=9 tane çözüm üçlüsü vardır.
Başlık: Ynt: 20. Ulusal Antalya Matematik Olimpiyatı Soruları
Gönderen: muuurat - Temmuz 07, 2015, 02:47:49 ös
16.
($\sqrt{3}+\sqrt{2}$)3=$9\sqrt{3}+11\sqrt{2}$
P(x)=(-1/2)x3+(11/2)x+1 polinomu aranan polinomdur.Buradan P(3)=4 bulunur.
Başlık: Ynt: 20. Ulusal Antalya Matematik Olimpiyatı Soruları
Gönderen: muuurat - Temmuz 08, 2015, 06:18:35 öö
14.
$ G $ Merkez ve $ |GC|=r $ olmak üzere $|CE|=x $ ve $|CF|=y $ olsun.$ r^2=xy $ ve Pisagor dan $ x^2+y^2=4r^2 $elde edilir.Buradan $ x^2-4xy+y^2=0 $ denklemi elde edilir.Denklemden $ x=y(2\mp\sqrt{3}) $ olur.Bu da bilindiği üzere $ 15^\circ,75^\circ,90^\circ $ üçgenidir.Yani en küçük açı $ 15^\circ $ dir.
SimplePortal 2.3.3 © 2008-2010, SimplePortal